Board logo

標題: 103中央大學附屬中壢高中 [打印本頁]

作者: bugmens    時間: 2014-4-26 17:19     標題: 103中央大學附屬中壢高中

美夢成真教甄討論文章
http://www.shiner.idv.tw/teachers/viewtopic.php?f=53&t=3291

[ 本帖最後由 bugmens 於 2014-4-26 09:56 PM 編輯 ]

附件: 103中央大學附屬中壢高中.pdf (2014-4-26 17:19, 175.24 KB) / 該附件被下載次數 19983
https://math.pro/db/attachment.php?aid=2142&k=3b30cc1d740b25355dd1cb4ed9e01c3c&t=1713539842

附件: 103中央大學附屬中壢高中答案.pdf (2014-4-26 17:19, 79.2 KB) / 該附件被下載次數 20157
https://math.pro/db/attachment.php?aid=2143&k=552cf4e708430a625bed5591e121476e&t=1713539842
作者: bugmens    時間: 2014-4-26 17:21

填充9.
ABCD-EFGH為一正立方體,各邊長為3,O為正立方體的中心,且\( \overline{EI}:\overline{IH}=2:1 \),\( \overline{DJ}:\overline{JH}=2:1 \),求「O,I,J三點所決定之平面」與「正立方體」所截的截面面積為。
[解答]
坐標化算出OIJ平面方程式以及和正立方體的交點
梯形的上底\( \overline{IJ}=\sqrt{1^2+1^2}=\sqrt{2} \)
\( \overline{LJ}=\sqrt{\overline{LC}^2+\overline{CD}^2+\overline{DJ}^2}=\sqrt{2^2+3^2+2^2}=\sqrt{17} \),梯形的高為\( \displaystyle \frac{\sqrt{17}}{2} \)
梯形的下底\( \overline{NK}=\sqrt{3^2+3^2}=3 \sqrt{2} \)
\( \displaystyle 六邊形IJKLMN面積=2 \times 梯形IJKN面積=2 \times \frac{\sqrt{17}}{2}\times \frac{\sqrt{2}+3 \sqrt{2}}{2}=2\sqrt{34} \)


103.5.25補充
以\( A(1,1,1) \),\( B(-1,1,1) \),\( C(-1,-1,1) \),\( D(1,-1,1) \),\( E(1,1,-1) \),\( F(-1,1,-1) \),\( G(-1,-1,-1) \),\( H(1,-1,-1) \)為頂點的正立方體。今有一平面\( x+2y+3z=4 \)與它相截,試問其截面面積為  
(93筆試二,臺灣師大數學系大學甄選入學指定項目甄試試題,http://www.math.ntnu.edu.tw/down/archive.php?class=105)

填充12.
設兩數列\( a_1,a_2, \ldots ,a_{100} \)及\( b_1,b_2, \ldots ,b_{100} \)滿足\( \displaystyle \cases{a_{n+1}=3a_n-2b_{n+1} \cr b_{n+1}=a_{n+1}-3b_n} \),\( n=1,2, \ldots ,99 \)。已知\( a_{99}=3^{50} \),\( b_{100}=4 \dots 3^{49} \)。試求\( \Bigg[\; \matrix{a_1 \cr b_1} \Bigg]\;= \)
(我的教甄準備之路-求數列一般項,https://math.pro/db/viewthread.php?tid=661&page=3#pid9507)


計算2.
\( \displaystyle \sqrt{\frac{1}{16}x^4-\frac{3}{2}x^2-6x+34}+\sqrt{\frac{1}{16}x^4+\frac{1}{2}x^2+1} \)的最小值為?
[解答]
\( \displaystyle \sqrt{(x-3)^2+(\frac{1}{4}x^2-5)^2}+\sqrt{(x-0)^2+(\frac{1}{4}x^2-1)^2} \)
可以看成\( A(3,5) \),\( B(0,1) \),P為\( \displaystyle y=\frac{1}{4}x^2 \)上的一個動點,要找\( \overline{PA}+\overline{PB} \)的最小值?

當APB三點共線時,有最小值\( \overline{AB}=5 \)←此行有錯
請參閱https://math.pro/db/viewthread.php?tid=1868&page=3#pid10086
感謝wen0623,一心老師指教


比較常見的是這兩題
試求\( f(x)=\sqrt{x^4-3x^2-6x+13}-\sqrt{x^4-x^2+1} \)之最大值為何?
(1992大陸高中數學競賽,91中一中段考題,95基隆高中,高中數學101 P235)

求函數\( f(x)=\sqrt{x^4-3x^2+4}+\sqrt{x^4-3x^2-8x+20} \)的最小值?
(88全國高中數學競賽 台北市,95台中高農,96彰師附工,97文華高中)

[ 本帖最後由 bugmens 於 2014-5-25 08:22 AM 編輯 ]

附件: 截面面積SketchUp檔.zip (2014-4-26 20:42, 42.92 KB) / 該附件被下載次數 18469
https://math.pro/db/attachment.php?aid=2145&k=1647cb23f8bf199fbd9ea63caa260e0b&t=1713539842

圖片附件: 截面面積.gif (2014-4-26 20:58, 18.86 KB) / 該附件被下載次數 9673
https://math.pro/db/attachment.php?aid=2146&k=6fd9e3f488a7d99b2415947230c9a6ec&t=1713539842


作者: shingjay176    時間: 2014-4-26 19:24     標題: 回復 2# bugmens 的帖子

這張考卷,我有去考。等等訂正把答案貼出來。讓我最嘔的是,填充題第十一題,最近算寸絲的講義,算了第200題,前面題目有遇到類似取高斯函數的題目,方法也會了,考場上算出四個答案。我只驗算真數要恆正,沒有驗算原來的等式。因此我的答案寫了四個,包含那個正確答案。不知道這題可以撿到幾分~~~

11、若實數 \(x\) 滿足 \({{\left( \log x \right)}^{2}}-\left[ \log x \right]-3=0\) ,則\(x\)=?  
\[\begin{array}{*{20}{l}}
{}&{\left[ {\log x} \right] = t \Rightarrow t \le \log x < t + 1}\\
{}&{ \Rightarrow \left( {\log x} \right) - 1 < t \le \log x}\\
{}&{{{\left( {\log x} \right)}^2} - 3 = \left[ {\log x} \right] = t}\\
{}&{ \Rightarrow \left( {\log x} \right) - 1 < {{\left( {\log x} \right)}^2} - 3 \le \log x}\\
{}&\begin{array}{l}
\log x = A\\
A - 1 < {A^2} - 3 \le A\\
\Rightarrow \left\{ \begin{array}{l}
A - 1 < {A^2} - 3\\
{A^2} - 3 \le A
\end{array} \right. \Rightarrow \left\{ \begin{array}{l}
{A^2} - A - 2 > 0\\
{A^2} - A - 3 \le 0
\end{array} \right. \Rightarrow \left\{ \begin{array}{l}
A > 2,A <  - 1\\
- 1. \cdots  \simeq \frac{{1 - \sqrt {13} }}{2} \le A \le \frac{{1 + \sqrt {13} }}{2} \simeq 2. \cdots
\end{array} \right.\\
\Rightarrow \frac{{1 - \sqrt {13} }}{2} \le A <- 1   ,    2 < A \le \frac{{1 + \sqrt {13} }}{2}
\end{array}
\end{array}\]
\[\left[ {\log x} \right] =  - 2,[\log x] = 2\]

帶回原式 ,
(1)  \(\begin{array}{l}
{\left( {\log x} \right)^2} - ( - 2) - 3 = 0\\
\Rightarrow \log x =  \pm 1 \Rightarrow x = 10 \vee x = \frac{1}{{10}}
\end{array}\)

(2) \(\begin{array}{l}
{\left( {\log x} \right)^2} - (2) - 3 = 0\\
\Rightarrow \log x =  \pm \sqrt 5  \Rightarrow x = {10^{\sqrt 5 }} \vee x = {10^{ - \sqrt 5 }}
\end{array}\)

\(x = 10,\frac{1}{{10}},{10^{\sqrt 5 }},{10^{ - \sqrt 5 }}\)
帶回 \({{\left( \log x \right)}^{2}}-\left[ \log x \right]-3=0\)  驗算

驗算發現只有 \({10^{\sqrt 5 }}\),符合等式。其餘都不合。另外三個答案,我在寫的時候,只驗算是否真數恆正。
沒有想到驗算這個原本題目的等式。因此四個答案都寫下去了。應該是都沒有分數了。(~~~樂極生悲,可惜了~~~會寫的題目就要步步驚心,小心把正確答案找出來)~~

[ 本帖最後由 shingjay176 於 2014-4-26 09:12 PM 編輯 ]
作者: tacokao    時間: 2014-4-26 20:27     標題: 回復 3# shingjay176 的帖子

可以用勘根定理就好~就可以知道[logx]=2了~
作者: tsusy    時間: 2014-4-26 20:36     標題: 回復 4# tacokao 的帖子

非連續函數
作者: wen0623    時間: 2014-4-27 12:02     標題: 回復 2# bugmens 的帖子

計算題第二題,也是這樣算,

最小值算出來是5

但答案怎是6?
作者: shingjay176    時間: 2014-4-27 13:02     標題: 回復 3# shingjay176 的帖子

1、在1到100之間的正整數n中,使得\({{n}^{2}}+7\) 與\(n+4\) 不互質的n有幾個? (這題目我在考場上,看到的第一題,又是今年第一家筆試,正個沒有了解題目意思,當下當成1到100中有多少個正整數與\({{n}^{2}}+7\) 和\(n+4\)不互質~~難怪當下越想越奇怪,整個沒有了解題目意思。犯了學生常犯的錯誤)

\({{n}^{2}}+7\) 與\(n+4\) 不互質,代表\({{n}^{2}}+7\) 與\(n+4\) 最大公因數不是1。
因此使用輾轉相除法。

\({n^2} + 7\)與\(n+4\),最大公因數23  
令 \({n^2} + 7=23h\)
    \(n+4=23k\)               \((h,k)=1\)

\(\begin{array}{l}
1 \le n = 23k - 4 \le 100\\
\Rightarrow 0. \cdots  \le k \le 4. \cdots \\
k = 1,2,3,4
\end{array}\)

共有4個

2、設\(f(x) = 2{x^5} + 3{x^4} + 3{x^3} + 5{x^2} + 6x + 10\),則\(f(96) \div 193\)的餘數為?

觀察發現\(193=(96)(2)+1\),因此把 \(f(x) = 2{x^5} + 3{x^4} + 3{x^3} + 5{x^2} + 6x + 10\)除以 \(2x+1\)
使用綜合除法
\(f(x) = 2{x^5} + 3{x^4} + 3{x^3} + 5{x^2} + 6x + 10 = (2x + 1)Q(x) + r\),求出 \(r=8\)
\(\begin{array}{l}
f(x) = (2x + 1)Q(x) + 8\\
f(96) = (2 \times 96 + 1)Q(96) + 8
\end{array}\)

答案  \(8\)

3、設 \(f(x) = {x^2} + 2x - 3, - 4 \le x \le 1\),求合成函數 \(f\left( {f\left( {f\left( x \right)} \right)} \right)\)之最大值為?

\(\begin{array}{l}
y = f(x) = {x^2} + 2x - 3, - 4 \le x \le 1\\
y = f\left( x \right) = {\left( {x + 1} \right)^2} - 4, - 4 \le x \le 1
\end{array}\)
這是一個開口向上的拋物線,頂點會產生最小值。頂點的\(x\)座標有包含在範圍內,可以得到
\[ \Rightarrow  - 4 \le y \le 5\]
\[\begin{array}{l}
f\left( {f\left( {f\left( x \right)} \right)} \right) = f\left( {f\left( y \right)} \right)\\
k = f\left( y \right) = {\left( {y + 1} \right)^2} - 4, - 4 \le y \le 5\\
\Rightarrow  - 4 \le k \le 32\\
f\left( k \right) = {\left( {k + 1} \right)^2} - 4, - 4 \le k \le 32
\end{array}\]

當k=32時,原來題目的合成函數有最大值1085


填充題第7題,就看13樓,寸絲老師的解法

[ 本帖最後由 shingjay176 於 2014-4-27 08:10 PM 編輯 ]
作者: yachine    時間: 2014-4-27 13:47     標題: 第六題

各位 填充6  不覺得討論出來的東西很怪嗎?

光log的真數的自然限制 就只能是上半圓了

可是底數的x+y+1不需要討論0<x+y+1<1  和 x+y+1>1 ?
如果在x+y+1>1的情況下 討論到    x^2+y^2<=1
那圖形切出來 會是135度的扇形耶
作者: shiauy    時間: 2014-4-27 13:58

計算2.
答案是6無誤,最小值5並不會發生,因為AB線段並不與拋物線相交
原式配方後看成P(x,y)至A點距離+P(x,y)至準線y=-1的距離和
最小值發生在PA直線與準線垂直
故答案為5+1=6
作者: tsusy    時間: 2014-4-27 14:07     標題: 回復 8# yachine 的帖子

填充 6 沒有很怪,圖形如下,虛實線未標



圖片附件: 2014.04.27.中壢高中6.png (2014-4-27 14:07, 5.91 KB) / 該附件被下載次數 10705
https://math.pro/db/attachment.php?aid=2151&k=0e7ff9794a715055b1091962334cd3f3&t=1713539842


作者: yachine    時間: 2014-4-27 14:10     標題: 回復 10# tsusy 的帖子

哈哈    原來 我少考慮到左邊圓外
謝謝你
作者: yachine    時間: 2014-4-27 14:12     標題: 填充7

請問大家怎麼考量呢
我想到的是先假設S=原式
之後乘1/16 去相減

後續卡住 XD
作者: tsusy    時間: 2014-4-27 14:39     標題: 回復 12# yachine 的帖子

填7. 乘 \( \frac14 \) 相減就好了,但是這個操作要做兩次

由 Ratio Test,知該級數收斂,令 \( S=\sum\limits _{n=2}^{\infty}\frac{n^{2}-1}{4^{n}} \)

則 \( (1-\frac{1}{4})S=\frac{3}{16}+\sum\limits _{n=3}^{\infty}\frac{2n-1}{4^{n}} \Rightarrow\frac{3}{4}S=\frac{3}{16}+\sum\limits _{n=3}^{\infty}\frac{2n-1}{4^{n}} \)

\( \Rightarrow(1-\frac{1}{4})(\frac{3}{4}S)=\frac{3}{4}\cdot\frac{3}{16}+\frac{5}{4^{3}}+\sum\limits _{n=4}^{\infty}\frac{2}{4^{n}} \)

\( \Rightarrow\frac{9}{16}S=\frac{9}{64}+\frac{5}{64}+\frac{2}{256}\cdot\frac{1}{1-\frac{1}{4}}=\frac{11}{48}
  \Rightarrow S=\frac{11}{48}\cdot\frac{16}{9}=\frac{11}{27} \)。
作者: perfectcrazy    時間: 2014-4-27 16:36

大家都好強…
好厲害…
:-D
作者: shingjay176    時間: 2014-4-27 16:46     標題: 回復 14# perfectcrazy 的帖子

第四題
設\(ABCD\)為矩形,\(\overline {AB}  = 1,\overline {BC}  = 2,P\)為射線\(\overrightarrow {BC} \)上一點,使\(\tan \left( {\angle APC} \right) = \frac{1}{3}\),求\(\overline {PD} \)長為?
(我先說我的想法,在考場時候,這個題目很容易思考到座標化,如下圖。可以算出直線 \(BD\)的方程式\(y = \frac{1}{2}x\),然後假設\(p\)點的參數式,\(p(2t,t),t \ge 0\),由於\(\tan \left( {\angle APC} \right) = \frac{1}{3}\)的值為正,所以這個角度為銳角,因此\(p\)會落在矩形外面,如下圖的參考圖形。我想藉由向量內積。向量\(PA\),向量\(PC\)內積去列出一個等式。解出 \(t\)這個未知數。列出來的等式如下
\[\sqrt {{{\left( { - 2t} \right)}^2} + {{\left( {1 - t} \right)}^2}}  \times \sqrt {{{\left( {2 - 2t} \right)}^2} + {{\left( { - t} \right)}^2}}  \times \frac{3}{{\sqrt {10} }} = 5{t^2} - 3t\]
如果要解出\(t\),兩邊一平方後,就變成4次的方程式,而且係數很大。想法可以,但算不出來。在考場上就掛住了~~~

剛剛自己訂正又想到用三角函數的方法去做,答案有算出來了。

\(\begin{array}{l}
\angle APC = \theta  = \theta 2 + \theta 1,\tan \theta  = \frac{1}{3}\\
\Rightarrow \left( {\theta 2 + \theta 1} \right) + \left( {\theta 3 + \theta 4} \right) = {90^0}\\
\Rightarrow \theta  = {90^0} - \left( {\theta 3 + \theta 4} \right)\\
\frac{1}{3} = \tan \theta  = \tan \left( {{{90}^0} - \left( {\theta 3 + \theta 4} \right)} \right) = \cot \left( {\theta 3 + \theta 4} \right) = \frac{1}{{\tan \left( {\theta 3 + \theta 4} \right)}}
\end{array}\)

\[\frac{1}{3} = \frac{1}{{\tan \left( {\theta 3 + \theta 4} \right)}}\]
\[ \Rightarrow \frac{1}{3} = \frac{{1 - (\tan \theta 3)(\tan \theta 4)}}{{(\tan \theta 3) + (\tan \theta 4)}}\]

\[ \Rightarrow \frac{1}{3} = \frac{{1 - \frac{{2t - 2}}{t} \times \frac{{t - 1}}{{2t}}}}{{\frac{{2t - 2}}{t} + \frac{{t - 1}}{{2t}}}}\]

\[ \Rightarrow \frac{{5{t^2} - 5t}}{{2{t^2}}} = \frac{{6{t^2} - 6{t^2} + 12t - 6}}{{2{t^2}}}\]

因為\(t\)大於0,因此得到分子會相等。

\[5{t^2} - 5t = 6{t^2} - 6{t^2} + 12t - 6\]

解出來 \[t = 3 \vee \frac{2}{5}\]  \(t = \frac{2}{5}\)不合

由此可知\(p(6,3)\),就可以解出 \[\overline {PD}  = \sqrt {20} \]

[ 本帖最後由 shingjay176 於 2014-4-27 08:11 PM 編輯 ]

圖片附件: 圖片1.gif (2014-4-27 16:48, 15.52 KB) / 該附件被下載次數 5583
https://math.pro/db/attachment.php?aid=2152&k=7c5ef8841e28b25ee69e4fa0aca8bb0e&t=1713539842


作者: tsusy    時間: 2014-4-27 17:27

填 10. 這類的排組,每個人的方法都略有不同,提供一個方式

先慮丁戊兩人在前三和後三的分布為
(2,0) xoxooo,丁戊有 2 種排法,第一個 o,僅有兩人可站,故此類有 \( 2 \times 2 \times 3! = 24 \)
(1,1) xooxoo,丁戊有 \( (3+3+2)\times 2 =16 \) 種排法;甲乙丙三人恰一人只能排後兩個 o;排完後己有三個位置可選;己選完位後,甲乙丙另兩人恰剩 1 種排法,故此類有 \( 16 \times 2 \times 3 = 96 \)
(0,2) oooxox,丁戊有 2 種排法,己有 4 個位置可選,選後甲乙丙恰一種排法,故此類有 \( 2 \times 4 =8 \) 種

綜合以上,共有128 種排法。
作者: shingjay176    時間: 2014-4-27 17:35     標題: 回復 16# tsusy 的帖子

排列組合這題目,我當下直覺放棄。考慮很多可能,算了半天又不一定對。等等再來好好訂正算一次。

第五題  \(\Delta ABC\)中,已知\(\overline {BC}  = 4\),\(\vec{BC} \cdot \vec{CA} =2 \vec{CA} \cdot \vec{AB} =3 \vec{AB} \cdot \vec{BC} \)
求線段\(AC\)長度為?

\[ab\cos \left( {\pi  - C} \right) = 2bc\cos \left( {\pi  - A} \right) = 3ca\cos \left( {\pi  - B} \right)\]
\[ - ab\cos C =  - 2bc\cos A =  - 3ca\cos B\]
\[ab\frac{{{a^2} + {b^2} - {c^2}}}{{2ab}} = 2bc\frac{{{b^2} + {c^2} - {a^2}}}{{2bc}} = 3ca\frac{{{c^2} + {a^2} - {b^2}}}{{2ca}}\]
\[{a^2} + {b^2} - {c^2} = 2{b^2} + 2{c^2} - 2{a^2} = 3{c^2} + 3{a^2} - 3{b^2}\]
\[\left\{ \begin{array}{l}
{a^2} + {b^2} - {c^2} = 2{b^2} + 2{c^2} - 2{a^2}\\
{a^2} + {b^2} - {c^2} = 3{c^2} + 3{a^2} - 3{b^2}
\end{array} \right.\]
\[\left\{ \begin{array}{l}
- 2{b^2} - 6{c^2} =  - 6{a^2}\\
6{b^2} - 6{c^2} = 3{a^2}
\end{array} \right.\]
\[{b^2} = \frac{9}{8}{a^2} \Rightarrow b = \frac{3}{{2\sqrt 2 }} \times 4 = 3\sqrt 2 \]

[ 本帖最後由 shingjay176 於 2014-4-27 07:31 PM 編輯 ]

圖片附件: [第五題參考圖] 圖片1_副本.png (2014-4-27 17:44, 47.3 KB) / 該附件被下載次數 5524
https://math.pro/db/attachment.php?aid=2155&k=6d400cb8a1b656a637ef589131ef5cfb&t=1713539842


作者: chin    時間: 2014-4-27 19:26     標題: 計算二

用微分=0可求出n=3,6,-2/3
當n=3,所求有min=6
作者: shingjay176    時間: 2014-4-27 19:33     標題: 回復 17# shingjay176 的帖子

填充題第六題
求\(\log _{\left( {x + y + 1} \right)}^{}\sqrt {1 - {x^2}}  \ge \log _{\left( {x + y + 1} \right)}^{}y\)的圖形面積為?

(1)  此時取出的範圍是在圓內
\[\left\{ \begin{array}{l}
x + y + 1 > 1\\
\sqrt {1 - {x^2}}  > 0\\
y > 0\\
\sqrt {1 - {x^2}}  \ge y
\end{array} \right. \Rightarrow \left\{ \begin{array}{l}
x + y > 0\\
- 1 < x < 1\\
y > 0\\
{x^2} + {y^2} \le 1
\end{array} \right.\]

(2) 此時取出的範圍是在圓內
\[\left\{ \begin{array}{l}
0 < x + y + 1 < 1\\
\sqrt {1 - {x^2}}  > 0\\
y > 0\\
\sqrt {1 - {x^2}}  \ge y
\end{array} \right. \Rightarrow \left\{ \begin{array}{l}
0 < x + y + 1 < 1\\
- 1 < x < 1\\
y > 0\\
{x^2} + {y^2} \ge 1
\end{array} \right.\]

\[\left( {\frac{1}{2} \times {{\left( 1 \right)}^2} \times \left( {\frac{{3\pi }}{4}} \right)} \right) + \left\{ {\frac{1}{2} \times 1 \times 1 - \frac{1}{2}{{\left( 1 \right)}^2} \times \frac{\pi }{4}} \right\} = \frac{1}{2} + \frac{\pi }{4}\]  
答案  \[\frac{1}{2} + \frac{\pi }{4}\]

[ 本帖最後由 shingjay176 於 2014-4-27 07:53 PM 編輯 ]

圖片附件: 103中壢高中教師甄選填充題第六題_副本_副本.jpg (2014-4-27 19:53, 450.98 KB) / 該附件被下載次數 5404
https://math.pro/db/attachment.php?aid=2157&k=1f3ee00e53573fdd66c522d65eb5cbfb&t=1713539842


作者: tsusy    時間: 2014-4-27 19:39     標題: 回復 18# chin 的帖子

不知道您怎麼微的,這個根號讓人不敢領教,只好由 WolframAlpha 代勞

differential sqrt(x^4/16-3/2x^2-6x+34)+sqrt(x^4/16+x^2/2+1)
solve 2 ((x+x^3/4)/sqrt((4+x^2)^2)+(-6-3 x+x^3/4)/sqrt(544-96 x-24 x^2+x^4))=0


solve 2 ((x+x^3/4)/sqrt((4+x^2)^2)+(-6-3 x+x^3/4)/sqrt(544-96 x-24 x^2+x^4))=0

做出來的結果,微分 =0,僅有 x=3,並非您寫的 3 個?
作者: idontnow90    時間: 2014-4-27 19:40

想請教第十題..我的做法哪裡錯了?謝謝~
用我是用"取捨"原理
為什麼我的照片貼不上去???
算了。。。用打的
我是用丁戊不相鄰-丁戊不相鄰的狀況下符合甲1或甲2或乙2或乙3或丙1或丙3的狀況
4!*5*4-6*3!*4*3+*2!*3*2-*1*2!=152
9指的是兩兩交集:甲1乙2。甲1乙3。甲1丙3。甲2乙3。甲2丙1。甲2丙3。乙2丙1。乙2丙3。乙3丙1共9種
2指的是三三交集:甲1乙2丙3。甲2乙3丙1共2種

[ 本帖最後由 idontnow90 於 2014-4-27 08:05 PM 編輯 ]
作者: shingjay176    時間: 2014-4-27 19:42     標題: 回復 20# tsusy 的帖子

確實,這個函數長相真的很醜。我不敢微分下手。
越容易思考的觀念,計算過程越複雜。
填充題第四題,一般來說看到三角形,就是餘弦定理或向量內積。我列出算式後。根本算不下去。
時間在催我~~
作者: shingjay176    時間: 2014-4-27 19:43     標題: 回復 21# idontnow90 的帖子

第十題我當下看去,要一個式子取捨定理寫出來,不太可能。
應該是要用分類討論的~~
可以把作法貼出來,這版上很多老師會幫你看看算式
作者: chin    時間: 2014-4-27 20:05     標題: 回復20#

我先將f(n)化簡再微分
f(n)=[√1/16(n^2-16)^2+1/2(n-16)^2]+(n^2+4)/4
f'=0得 (1/4n^3-3n-6)/[1/4√(n^2-16)^2+8(n-16)^2]+n=0
(n^3-12n-24)/[√(n^2-16)^2+8(n-16)^2]=-n
兩邊平方可得3n^3-25n^2+36n+36=0
                   即(n-3)(n-6)(3n+2)=0
作者: shingjay176    時間: 2014-4-27 20:09     標題: 回復 24# chin 的帖子

可以用寸絲老師部落格的教學,我現在都是用方程式編輯器,轉換成LATEX語法。
http://tsusy.wordpress.com/2013/05/05/latex-%E2%86%94-mathtype/

填充題第十題
甲乙丙丁戊己共六人排成一列,其中甲不排在第1,2位,乙不排在第2,3位置,丙不排在第1,3位置且丁戊不相鄰的排列有幾種?


註解:這個問題在考場上,我直覺用排容原理不好算,但這題直接去那樣排容,每個子問題還是不好做,變得跟分類討論沒有什麼兩樣。因此每種狀況詳細列出來,分類討論去算。會比較好思考。考試時候,我過往經驗排列組合,只要討論有漏,或是多算。時間花下去了,分數又沒拿到。很吃虧。
考場上我試著去討論排列甲乙丙的可能所有位置,但又要考慮丁戊相鄰與否。因此變得十分複雜。
這個題目只要考慮丁戊,就單純許多


[ 本帖最後由 shingjay176 於 2014-4-27 11:19 PM 編輯 ]

圖片附件: 填充題第十題_副本.png (2014-4-27 23:12, 49.04 KB) / 該附件被下載次數 4998
https://math.pro/db/attachment.php?aid=2159&k=cff9d55817dd3d4d85f639a60a7caf30&t=1713539842


作者: tsusy    時間: 2014-4-27 20:24     標題: 回復 24# chin 的帖子

第二、三、四行有個筆誤,根號裡的分母寫錯了應該是 \( (n^2-16)^2 + 8(n-\color{red}{6})^2 \) 才正確

第五行的式子是正確的,但平方的同時增根了
作者: chin    時間: 2014-4-27 20:30     標題: 回復25#,26#

啪勢啦!謝謝更正
因為在外面,用手機打的較慢又有筆誤
作者: tsusy    時間: 2014-4-27 20:38     標題: 回復 21# idontnow90 的帖子

您的計算中 -6*3!*4*3,
6先挑出甲乙丙其1人及丁戊並決定其位子(未排入),其餘3人排列 3!,丁戊再插入 \( 3! \times 4 \times 3\)
再把剛挑出的甲乙丙一人插入至其位子。

但這樣的子,會漏掉 丁甲戊乙丙己 這類 丁戊中恰為一開始被挑出的甲乙丙。
其它項亦同。

發生的問題點其實是在計算中用了兩種不相容的排法,選固定位子序直線排列自動生產位序
作者: wen0623    時間: 2014-4-27 20:48     標題: 回復 9# shiauy 的帖子

A,B均在開囗向上之抛物線內部,B為焦點,準線L:y=—1,線段BP=d(P,L),線段AP+線段BP=線段AP+d(P,L)>=d(A,L)=6,謝謝—心老師。~魔鬼果然藏在細節裡>_<。

[ 本帖最後由 bugmens 於 2014-4-27 09:22 PM 編輯 ]

圖片附件: 拋物線.gif (2014-4-27 21:20, 4.08 KB) / 該附件被下載次數 4880
https://math.pro/db/attachment.php?aid=2158&k=014ef3cf0dd71e14014187a79d64a43f&t=1713539842


作者: idontnow90    時間: 2014-4-28 00:03

謝謝寸絲老師.我知道我錯在哪了~

另外想請教計算1.
我這作法不知道是否正確.
用算幾.(a+2b)/2>=根號2ab..令X=ab
X^2-68X+900<=0
X<=18或 X>=50
所以max=18

[ 本帖最後由 idontnow90 於 2014-4-28 12:14 AM 編輯 ]
作者: shingjay176    時間: 2014-4-28 08:46

填充題第十二題,有何想法?從何下筆!
我只剩下這題還沒訂正出來答案。
作者: tsusy    時間: 2014-4-28 09:14     標題: 回復 31# shingjay176 的帖子

填 12. 把 \( a_{n+1}, b_{n+1} \) 當作未知數,做消去法,可得

\( \begin{bmatrix}a_{n+1}\\
b_{n+1}
\end{bmatrix}=\begin{bmatrix}1 & 2\\
1 & -1
\end{bmatrix}\begin{bmatrix}a_{n}\\
b_{n}
\end{bmatrix} \) ,而 \( \begin{bmatrix}1 & 2\\
1 & -1
\end{bmatrix}^{2}=3I_{2} \),故 \( a_{n+2}=3a_{n}
, b_{n+2}=3b_{n} \)。
作者: shingjay176    時間: 2014-4-28 09:22     標題: 回復 32# tsusy 的帖子

寸絲老師,謝謝。
作者: superlori    時間: 2014-4-28 11:18     標題: 回復 31# shingjay176 的帖子

給您參考看看

圖片附件: 壢中填充12.jpg (2014-4-28 11:18, 102.98 KB) / 該附件被下載次數 5272
https://math.pro/db/attachment.php?aid=2162&k=c78963146b118b6ce6bd8811d7d27a01&t=1713539842


作者: YAG    時間: 2014-4-28 11:21     標題: 回復 30# idontnow90 的帖子

過程錯誤 不然我也可以這樣啊

錯解:
(甲)
a + 2b ≧ 2√2ab
等號成立於 a = 2b 時,代入 a + 2b + ab = 30
2b^2 + 4b -30 = 0, b^2+2b-15=0
a = 6,b = 3  max ab =18
(乙)
a+ b ≧ √ab
等號成立於 a = b 時,代入 代入 a + 2b + ab = 30
b^2+3b-30=0 很明顯 b不會如甲中的 b=3

[ 本帖最後由 YAG 於 2014-4-28 11:24 AM 編輯 ]
作者: tsusy    時間: 2014-4-28 11:33     標題: 回復 35# YAG 的帖子

不不不 你的方法和 30# idontnow90 完全不同

30# idontnow90 沒有用到任何等號成立的事,只單純使用不等式論證出 \( ab \leq 18 \) 或 \( ab \geq 50 \) 不合。

也就是說 30# 中,除了最後一行的所以外,沒有其它錯誤。

所以 30# 之處,結論應改成 \( ab \) 有上界 18。

30# 樓僅需再驗證,等號成立,欲使 \( X = 18 \),過程中每個不等式均須為等號,

故 \( a=2b, ab=18 \) 解得 \( a=6, b=3 \) 恰可使 \( ab \) 達到上界18,故18 為其最大值

[ 本帖最後由 tsusy 於 2014-4-28 11:38 AM 編輯 ]
作者: YAG    時間: 2014-4-28 13:21

引用:
原帖由 tsusy 於 2014-4-28 11:33 AM 發表
不不不 你的方法和 30# idontnow90 完全不同

30# idontnow90 沒有用到任何等號成立的事,只單純使用不等式論證出 \( ab \leq 18 \) 或 \( ab \geq 50 \) 不合。

也就是說 30# 中,除了最後一行的所以外,沒有其它錯誤。
...
請問35 樓中的甲乙方法是錯在哪裡?如果教師甄試考這題,要如何說明其錯誤?謝謝!
作者: tsusy    時間: 2014-4-28 13:35     標題: 回復 37# YAG 的帖子

我比較喜歡的說法是 沒有地方正確...

除了算幾不等式會讓人聯想到極值以外,其它沒有一個步驟處理了和極值有關的事
1. \( f(x) \leq m \)
2. 存在 \( x_0 \) 使得 \( f(x_0) = m \)
若 m 滿足 1,2 這兩件事,我們稱 m 是 \( f \) 的最大值

這兩件事,在甲、乙兩種過程裡完全沒有任何一丁點出現
作者: Ellipse    時間: 2014-4-28 22:49

猜猜看這張大約幾分進複試
我猜70(+-5)分
作者: Ellipse    時間: 2014-4-29 10:12

引用:
原帖由 Ellipse 於 2014-4-28 10:49 PM 發表
猜猜看這張大約幾分進複試
我猜70(+-5)分
公佈了
62分進複試
沒有想像中高
可見大家都有機會~
作者: shingjay176    時間: 2014-4-29 13:13     標題: 回復 40# Ellipse 的帖子

這份考卷真的不難,200多人應考,取8名。62分就過關了。祝福那些過關的版友。
作者: Ellipse    時間: 2014-4-29 14:13

引用:
原帖由 shingjay176 於 2014-4-29 01:13 PM 發表
這份考卷真的不難,200多人應考,取8名。62分就過關了。祝福那些過關的版友。
若是我,考卷一發下來,會先從頭到尾看一遍
先勾會寫有把握的題目,沒看過沒把握會先放棄(這些題目大家也不會)
先把基本分拿到,再預測考卷幾分會進複試
想辦法把剩下不足的分數補齊~

[ 本帖最後由 Ellipse 於 2014-4-29 02:14 PM 編輯 ]
作者: lyingheart    時間: 2014-5-1 16:43     標題: 回復 15# shingjay176 的帖子

一開始也是用座標,然後內積,得到難看的四次方程式;
後來想想,P點所在位置,是對AC的張角固定,也就是在一個圓弧上。
假設此圓的圓心為O,AC和BD的交點為E,
那麼 \(\displaystyle \angle{AOE}=\angle{APC} \)

因為 \(\displaystyle AE=\frac{\sqrt{5}}{2} \) ,所以 \(\displaystyle OA=\frac{5\sqrt{2}}{2}, OE=\frac{3\sqrt{5}}{2} \)

那麼圓心座標就是 \(\displaystyle (\frac{5}{2},\frac{7}{2}) \),半徑就是OA,' 把圓方程式求出,求解與BD的交點就會得到那個比較簡單的方程式。

話說回來,有沒有哪個學校初試不用審查資料只要交錢就好的啊??
作者: Ellipse    時間: 2014-5-1 20:23

引用:
原帖由 lyingheart 於 2014-5-1 04:43 PM 發表

話說回來,有沒有哪個學校初試不用審查資料只要交錢就好的啊??
越來越多學校這樣做了,初試網路報名,轉帳或劃撥繳費
複試再審資料,以免考生舟車勞頓
這是未來的趨勢~~
作者: shingjay176    時間: 2014-5-1 21:17     標題: 回復 44# Ellipse 的帖子

沒錯,我100年考試,那年考了十七所。每一所都是現場報名。真的是超級累人。弄了半天又沒有過筆試。當初報名準備那些審查資料火大的。這兩三年已經變成網路報名。
筆試過關再到現場報名,審查資料
作者: smartdan    時間: 2014-5-1 21:21     標題: 回復 44# Ellipse 的帖子

目前新北聯招就是這個方式,
報名取得繳費資訊,繳完錢之後印准考證就可以去考試了,
進複試現場報名時再收資料。

不過現在大部分的學校報名初試還是要寄影本資料過去,
去複試的時候再核對正本,也是有少數學校還是堅持初試現場報名,
不過越來越便民真的是趨勢,對考生來說也是一件好事
作者: Ellipse    時間: 2014-5-1 21:44

引用:
原帖由 shingjay176 於 2014-5-1 09:17 PM 發表
沒錯,我100年考試,那年考了十七所。每一所都是現場報名。真的是超級累人。弄了半天又沒有過筆試。當初報名準備那些審查資料火大的。這兩三年已經變成網路報名。
筆試過關再到現場報名,審查資料 ...
早期曾有學校初試報名時,還要叫你交他規定的"自傳"
而且一定要手寫,還不能用電腦打,真是搞倒人仰馬翻~

那時也有學校複試時還要考資訊能力:word+excel+ppt
現在哪有學校在考這個...不會電腦又不代表不會教書~
當年那些學校真是的...複試:口試+試教+資訊能力
弄得考生好累...

還有講到年資,學歷(研究所)加分的問題,現在越來越多學校把這個資深的"福利"取消了
對剛畢業的老師是個公平的立足點~

[ 本帖最後由 Ellipse 於 2014-5-1 10:38 PM 編輯 ]
作者: lyingheart    時間: 2014-5-1 23:29     標題: 回復 44# Ellipse 的帖子

謝謝,那如果台北市有學校這樣甄試的話,麻煩通知一下,最近覺得腦袋已經糊塗了。
作者: wooden    時間: 2014-5-1 23:51

請教填充8:求四面體體積之總和
作者: tsusy    時間: 2014-5-2 08:44     標題: 回復 49# wooden 的帖子

填 8. 令 \( \vec{a}=\vec{OA_{k}}, \vec{b}=\vec{OB_{k}}, \vec{c}=\vec{OC_{k}} \),則 \( \vec{OA}_{k+1}=\frac{1}{3}(\vec{a}+\vec{b}), \vec{OB}_{k+1}=\frac{1}{3}(\vec{b}+\vec{c}), \vec{OC}_{k+1}=\frac{1}{3}(\vec{a}+\vec{c}) \) ,則有

\( \begin{vmatrix}\frac{1}{3}(\vec{a}+\vec{b})\\
\frac{1}{3}(\vec{b}+\vec{c})\\
\frac{1}{3}(\vec{c}+\vec{a})
\end{vmatrix}=\frac{1}{27}\begin{vmatrix}\vec{a}+\vec{b}\\
\vec{b}+\vec{c}\\
\vec{c}+\vec{a}
\end{vmatrix}=\frac{1}{27}\begin{vmatrix}2(\vec{a}+\vec{b}+\vec{c})\\
\vec{b}+\vec{c}\\
\vec{c}+\vec{a}
\end{vmatrix}=\frac{2}{27}\begin{vmatrix}\vec{a}+\vec{b}+\vec{c}\\
-\vec{a}\\
-\vec{b}
\end{vmatrix}=\frac{2}{27}\begin{vmatrix}\vec{c}\\
\vec{a}\\
\vec{b}
\end{vmatrix} \),故體積之公比為 \( \frac{2}{27} \) ,而首項為 \( v_{1}=\frac{1}{6}|\begin{vmatrix}3 & -1 & 2\\
1 & 2 & 3\\
-2 & -1 & 3
\end{vmatrix}|=7 \),

體積和為 \( \frac{7}{1-\frac{2}{27}}=\frac{189}{25} \)。
作者: wooden    時間: 2014-5-2 09:54     標題: 回復 50# tsusy 的帖子

感謝寸絲兄
作者: Ellipse    時間: 2014-5-2 22:09

引用:
原帖由 shingjay176 於 2014-4-27 04:46 PM 發表
第四題
設\(ABCD\)為矩形,\(\overline {AB}  = 1,\overline {BC}  = 2,P\)為射線\(\overrightarrow {BC} \)上一點,使\(\tan \left( {\angle APC} \right) = \frac{1}{3}\),求\(\overline {PD} \)長為?
(我先說我的想法, ...
乍看之下這題好像很麻煩
其實用課本定義就行了
(最基本的往往都忘了派上用場,看不起眼的公式,其實是最重要)
假設P(2t,t),A(0,1),B(0,0),C(2,0)
假設PC的斜率為m1 ,則m1=(t-0)/(2t-2)=t/(2t-2)
假設PA的斜率為m2 ,則m2=(t-1)/(2t-0)=(t-1)/(2t)
則tan(角APC)=(m1-m2)/(1+m1*m2) =1/3
解t

[ 本帖最後由 Ellipse 於 2014-5-2 10:13 PM 編輯 ]
作者: shingjay176    時間: 2014-5-2 22:20     標題: 回復 52# Ellipse 的帖子

確實,往往繞了一大圈,甚至最基本不起眼的公式。最好算。
就看當下想到甚麼公式觀念了。應該大家直覺都是內積或餘弦定理。
難道我們都被制約住了嗎?XD
作者: Ellipse    時間: 2014-5-2 22:25

引用:
原帖由 shingjay176 於 2014-5-2 10:20 PM 發表
確實,往往繞了一大圈,甚至最基本不起眼的公式。最好算。
就看當下想到甚麼公式觀念了。應該大家直覺都是內積或餘弦定理。
難道我們都被制約住了嗎?XD ...
是題目"角APC"讓人被困住在三角形APC內
以為用內積或餘弦定理
其實要轉成:直線AP與直限CP的夾角
用斜率&直線夾角公式來做
作者: tsusy    時間: 2014-5-3 15:09     標題: 回復 52# Ellipse 的帖子

填充 4. 坐標硬解法,注意稱性,將矩形的中心點取為原點,令\( A( - 1,\frac{1}{2}),C(1, - \frac{1}{2}),P(2t,t) \)

\( \vec{AP}\cdot \vec{CP} = 5{t^2} - \frac{5}{4} = \sqrt {5{t^2} + 3t + \frac{5}{4}}  \cdot \sqrt {5{t^2} - 3t + \frac{5}{4}}  \cdot \frac{{\sqrt 3 }}{{\sqrt {10} }}\)

\( \Rightarrow 25{(4{t^2} - 1)^2} = (20{t^2} + 12t + 5)(20{t^2} - 12t + 5) \cdot \frac{9}{{10}}\)

\( \Rightarrow 400{t^4} - 2504{t^2} + 25 = 0 \)

公式解可得 \( {t^2} = \frac{{2504 \pm 2596}}{{800}} = \frac{{25}}{4},\frac{1}{{100}} \Rightarrow t =  \pm \frac{5}{2}, \pm \frac{1}{{10}} \)

其中 \( \pm \frac1{10} \) 檢驗應為 \( \cos \alpha = - \frac{\sqrt{3}}{\sqrt{10}} \)
作者: panda.xiong    時間: 2014-5-29 11:50

請問填充第五題有沒有甚麼好方法?
作者: shingjay176    時間: 2014-5-29 12:00

17樓,我已經解過填充題第五題了
引用:
原帖由 panda.xiong 於 2014-5-29 11:50 AM 發表
請問填充第五題有沒有甚麼好方法?

作者: Pacers31    時間: 2014-5-29 19:24     標題: 回復 56# panda.xiong 的帖子

第5題:來個數學101的解法

設 \(\overset{\rightarrow}{BC}\cdot\overset{\rightarrow}{CA}=2\overset{\rightarrow}{CA}\cdot\overset{\rightarrow}{AB}=3\overset{\rightarrow}{AB}\cdot\overset{\rightarrow}{BC}=t\)

\(\overline{BC}=4\)    \(\Rightarrow\ \overset{\rightarrow}{BC}\cdot\overset{\rightarrow}{BC}=16\)    \(\Rightarrow\ \overset{\rightarrow}{BC}\cdot(\overset{\rightarrow}{AC}-\overset{\rightarrow}{AB})=16\)

\(\Rightarrow\ \displaystyle -t-\frac{t}{3}=16\)    \(\Rightarrow\ t=-12\)

\(\displaystyle \overline{AC}=\sqrt{\overset{\rightarrow}{AC}\cdot\overset{\rightarrow}{AC}}=\sqrt{\overset{\rightarrow}{AC}\cdot(\overset{\rightarrow}{BC}-\overset{\rightarrow}{BA})}=\sqrt{-t-\frac{t}{2}}=\sqrt{18}\)

[ 本帖最後由 Pacers31 於 2014-5-29 08:06 PM 編輯 ]
作者: martinofncku    時間: 2014-7-10 14:25     標題: 回復 7# shingjay176 的帖子

想請問老師, 填充 1 ,為什麼 \({{n}^{2}}+7\) 與\(n+4\)  最大公因數是 23?
作者: shingjay176    時間: 2014-7-10 14:30     標題: 回復 59# martinofncku 的帖子

輾轉相除法就可以求出
作者: martinofncku    時間: 2014-7-10 15:02     標題: 回復 60# shingjay176 的帖子

不好意思, 我比較鈍, \( n^{2}=(n+4)(n-4)=23 \)到這邊我就算不下去了...
作者: shingjay176    時間: 2014-7-10 16:08     標題: 回復 61# martinofncku 的帖子

如附件

[ 本帖最後由 shingjay176 於 2014-7-11 09:15 AM 編輯 ]

圖片附件: 輾轉相除法.gif (2014-7-11 00:32, 7.25 KB) / 該附件被下載次數 5728
https://math.pro/db/attachment.php?aid=2468&k=58b94aa6f02616d43d1e66ce9593d672&t=1713539842


作者: martinofncku    時間: 2014-7-10 19:26     標題: 回復 62# shingjay176 的帖子

請問老師,如何再把 n+4 除以 23 (的原理)
作者: shingjay176    時間: 2014-7-10 19:37     標題: 回復 62# shingjay176 的帖子

若\(x+1\)是\(f(x)\)與\(g(x)\)的公因式,則 \(d\)不可為0,\(d(x+1)\)也是這兩個多項式的公因式。因此兩多項式在做輾轉相除法,求 HCF時候,可以調整多項式的領導係數。方便計算。
因此我把多項式\(n+4\)  放大,前面乘上 \(23\)
作者: arend    時間: 2014-8-9 01:36

引用:
原帖由 tsusy 於 2014-5-2 08:44 AM 發表
填 8. 令 \( \vec{a}=\vec{OA_{k}}, \vec{b}=\vec{OB_{k}}, \vec{計算出錯c}=\vec{OC_{k}} \),則 \( \vec{OA}_{k+1}=\frac{1}{3}(\vec{a}+\vec{b}), \vec{OB}_{k+1}=\frac{1}{3}(\vec{b}+\vec{c}), \vec{OC}_{k+1}=\frac{1}{3 ...
請教寸絲老師
矩陣中第一列vec(a)+vec(b)=2(vec(a)+vec(b)+vec(c)) 怎麼來的?  
又vec(a)+vec(b)+vec(c)=0?
這我不通,不過這方法很漂亮
我自己是先求點座標,雖然答案一樣,但繁瑣多了,且考試中也容易出錯

謝謝
作者: superlori    時間: 2014-8-10 20:17     標題: 回復 65# arend 的帖子

(1)這是行列式的列運算,把第二列還有第三列加到第一列
(2)vec(a)+vec(b)+vec(c)=0→也沒有這個
他全部加到第一列之後把2倍提出去,然後再把第一列加到第二列第三列

從頭到尾都是行列式的列運算。

[ 本帖最後由 superlori 於 2014-8-10 08:35 PM 編輯 ]
作者: arend    時間: 2014-8-10 22:22

引用:
原帖由 superlori 於 2014-8-10 08:17 PM 發表
(1)這是行列式的列運算,把第二列還有第三列加到第一列
(2)vec(a)+vec(b)+vec(c)=0→也沒有這個
他全部加到第一列之後把2倍提出去,然後再把第一列加到第二列第三列

從頭到尾都是行列式的列運算。 ...
謝謝, 我一直在幾何方向打轉
作者: YAG    時間: 2014-8-30 21:14     標題: 回復 7# shingjay176 的帖子

f(x)=x^2+2x-3,求f(f(f(x))),-4<=x<=1的最值 。
這個題目沒限制f的定義域,那麼就是實數集,要求的是fff(x)在負4到1上的最大值 。答案是1085
f(x)=x^2+2x-3,-4<=x<=1,求f(f(f(x)))的最大值。
這個題目是首先限定了f(x)的定義域,那麼每一個f都要在-4到1內來操作,超出了範圍就沒意義 。答案是5
請問大家的看法是如何?




歡迎光臨 Math Pro 數學補給站 (https://math.pro/db/) 論壇程式使用 Discuz! 6.1.0